Sie sind auf Seite 1von 13

Problem

Let , , and all exceed , and let be a positive number such that , ,

and . Find .

Solutions
Solution 1

The logarithmic notation doesn't tell us much, so we'll first convert everything to the equivalent

exponential expressions.

, , and . If we now convert everything to a power of , it will be easy to

isolate and .

, , and .

With some substitution, we get and .

Solution 2

Applying the change of base formula,

Therefore, .

Hence, .

Problem
Let , where . Determine the minimum value taken

by for in the interval .

Solution
It is best to get rid of the absolute value first.

Under the given circumstances, we notice that , ,

and .

Adding these together, we find that the sum is equal to , of which the minimum value is attained

when .
Edit: can equal or (for example, if and

, ). Thus, our two "cases" are (if ) and (if ).

However, both of these cases give us as the minimum value for , which indeed is the answer

posted above.

Problem

What is the product of the real roots of the equation ?

Solution

If we expand by squaring, we get a quartic polynomial, which isn't very helpful.

Instead, we substitute for and our equation becomes .

Now we can square; solving for , we get or . The second solution is extraneous

since is positive. So, we have as the only solution for .

Substituting back in for ,

By Vieta's formulas, the product of the roots is .

Problem

A machine shop cutting tool is in the shape of a notched circle, as shown. The radius of the circle

is cm, the length of is 6 cm, and that of is 2 cm. The angle is a right angle. Find

the square of the distance (in centimeters) from to the center of the circle.

Solution
Solution 1
Because we are given a right angle, we look for ways to apply the Pythagorean Theorem. Let the foot of

the perpendicular from to be and let the foot of the perpendicular from to the line be .

Let and . We're trying to find .

Applying the Pythagorean Theorem, and .

Thus, , and . We solve this system to

get and , resulting in .

Solution 2

Drop perpendiculars from to ( ), to ( ), and to ( ). Also, draw the

midpoint of .

Then the problem is trivialized. Why?

First notice that by computation, is a isosceles triangle; thus . Then,

notice that . Thus the two blue triangles are congruent.


So, . As , we subtract and get . Then the

Pythagorean Theorem shows .

Problem
Let equal . Determine the remainder upon dividing by .

Solution
Solution 1

First, we try to find a relationship between the numbers we're provided with and . We realize

that and both and are greater or less than by .

Expressing the numbers in terms of , we get .

Applying the Binomial Theorem, half of our terms cancel out and we are left

with . We realize that all of these terms are divisible by except the

final term.

After some quick division, our answer is .

Solution 2

Since (the Euler's totient function), by Euler's Totient

Theorem, where .

Thus .

Alternatively, we could have noted that . This way, we


have , and can finish the same way.

Problem
Twenty five of King Arthur's knights are seated at their customary round table. Three of them are chosen -

all choices being equally likely - and are sent of to slay a troublesome dragon. Let be

the probability that at least two of the three had been sitting next to each other. If is written as a

fraction in lowest terms, what is the sum of the numerator and the denominator?

Solution
Solution 1

We can use Complementary counting by finding the probability that none are sitting next to each other

and subtracting it from .

Imagine the other (indistinguishable) people are already seated, and fixed into place.
We will place , , and with and without the restriction.

There are places to place , followed by places to place , and places to place after and .

Hence, there are ways to place in between these people with restrictions.

Without restrictions, there are places to place , followed by places to place , and places to

place after and . Hence, there are ways to place in between these people

without restrictions.

Thus, the desired amount is , and the answer

is .

Solution 2

There are configurations for the knights about the table.

There are ways to pick a pair of knights from the trio, and there are ways to determine

which order they are seated. Since these two knights must be attached, we let them be a single entity, so

there are configurations for the entities.

However, this overcounts the instances in which the trio sits together; when all three knights sit together,

then two of the pairs from the previous case are counted. However, we only want to count this as one

case, so we need to subtract the number of instances in which the trio sits together (as a single entity).

There are ways to determine their order, and there are configurations.

Thus, the answer is , and the answer is .

Solution 3

Number the knights around the table 1-25. There are two possibilities: All three sit next to each other, or

two sit next to each other and one is not sitting next to the other two.

Case 1: All three sit next to each other. In this case, you are picking , , ... .

This makes combinations.

Case 2: Like above, there are ways to pick the pair of knights sitting next to each other. Once a pair is

picked, you cannot pick either of the two adjacent knights. (i.e. if you pick , you may not pick 4 or

7). Thus, there are ways to pick the third knight, for a total of combinations.

Thus, you have a total of allowable ways to pick the knights. The total number of

ways to pick the knights is .


The probability is , and the answer is .

Solution 4

Pick an arbitrary spot for the first knight. Then pick spots for the next two knights in order.

Case 1: The second knight sits next to the first knight. There are 2 possible places for this out of 24, so

the probability of this is . We do not need to consider the third knight.

Case 2: The second knight sits two spaces from the first knight. There are 2 possible places for this out of

24, so the probability is . Then there are 3 places out of a remaining 23 for the third knight to sit, so

the total probability for this case is

Case 3: The second knight sits 3 or more spaces from the first knight. There are 20 possible places for

this out of 24, so the probability is . Then there are four places to put the last knight out of 23, so the

total probability for this case is

So add the probabilities to get the total:

Problem

What is the largest 2-digit prime factor of the integer ?

Solution

Expanding the binomial coefficient, we get . Let the prime be ; then .

If , then the factor of appears twice in the denominator. Thus, we need to appear as a factor

three times in the numerator, or . The largest such prime is , which is our answer.

Problem

Find the minimum value of for .

Solution
Solution 1
Let . We can rewrite the expression as .

Since and because , we have . So we can apply AM-GM:

The equality holds when .

Therefore, the minimum value is (when ; since is continuous and increasing on

the interval and its range on that interval is from , by the Intermediate Value

Theorem this value is attainable).

Solution 2

We can rewrite the numerator to be a perfect square by adding . Thus, we must also add

back .

This results in .

Thus, if , then the minimum is obviously . We show this possible with the same

methods in Solution 1; thus the answer is .

Solution 3

Let and rewrite the expression as , similar to the previous solution. To

minimize , take the derivative of and set it equal to zero.

The derivative of , using the Power Rule, is

is zero only when or . It can further be verified that and are relative minima by

finding the derivatives of other points near the critical points. However, since is always positive in

the given domain, . Therefore, = , and the answer is .


Problem
The solid shown has a square base of side length . The upper edge is parallel to the base and has

length . All other edges have length . Given that , what is the volume of the solid?

Solution
Solution 1

First, we find the height of the figure by drawing a perpendicular from the midpoint of to .

The hypotenuse of the triangle is themedian of equilateral triangle , and one of the legs is . We

apply the Pythagorean Theorem to find that the height is equal to .


Next, we complete the figure into a triangular prism, and find the volume, which is .

Now, we subtract off the two extra pyramids that we included, whose combined volume

is .

Thus, our answer is .

Solution 2

Extend and to meet at , and and to meet at . Now, we have a regular

tetrahedron , which has twice the volume of our original solid. This tetrahedron has side

length . Using the formula for the volume of a regular tetrahedron, which is ,

where S is the side length of the tetrahedron, the volume of our original solid is:

Problem
The length of diameter is a two digit integer. Reversing the digits gives the length of a perpendicular

chord . The distance from their intersection point to the center is a positive rational number.

Determine the length of .

Solution
Let and . It follows

that and . Applying the Pythagorean

Theorem on and

, .

Because is a positive rational number, the quantity cannot contain any square

roots. Either or must be 11. However, cannot be 11, because both must be digits.

Therefore, must equal eleven and must be a perfect square (since ). The only

pair that satisfies this condition is , so our answer is .

Problem
For and each of its non-empty subsets, an alternating sum is defined as follows. Arrange

the number in the subset in decreasing order and then, beginning with the largest, alternately add and

subtract succesive numbers. For example, the alternating sum

for is and for it is simply . Find the sum of all such alternating

sums for .

Solution 1
Let be a non- empty subset of .
Then the alternating sum of plus the alternating sum of with 7 included is 7. In mathematical

terms, . This is true because when we take an alternating sum, each term of has the

opposite sign of each corresponding term of .

Because there are of these pairs, the sum of all possible subsets of our given set is . However,

we forgot to include the subset that only contains , so our answer is .

Solution 2
Consider a given subset of that contains 7; then there is a subset which contains all the elements

of except for 7, and only those. Since each element of has one element fewer preceding it than it

does in , their signs are opposite; so the sum of the alternating sums of and is equal to 7. There

are subsets containing 7, so our answer is .

Problem
In the adjoining figure, two circles with radii and are drawn with their centers units apart. At , one

of the points of intersection, a line is drawn in such a way that the chords and have equal length.

( is the midpoint of ) Find the area of the square with a side length of .

Solution
Solution 1

First, notice that if we reflect over we get . Since we know that is on circle and is on

circle , we can reflect circle over to get another circle (centered at a new point with radius )

that intersects circle at . The rest is just finding lengths:

Since is the midpoint of segment , is a median of triangle . Because we know

that , , and , we can find the third side of the triangle using Stewart's

Theorem or similar approaches. We get . So now we have a kite with


, , and , and all we need is the length of the other diagonal . The easiest

way it can be found is with the Pythagorean Theorem. Let be the length of . Then

Doing routine algebra on the above equation, we find that , so

Solution 2

Draw additional lines as indicated. Note that since triangles and are isosceles, the altitudes

are also bisectors, so let .

Since triangles and are similar. If we let , we

have .

Applying the Pythagorean Theorem on triangle , we have . Similarly, for

triangle , we have .

Subtracting, .

Solution 3
Let . Angles , , and must add up to . By the Law of

Cosines, . Also, angles and equal and . So we


have

Taking the of both sides and simplifying using the cosine addition identity gives .

Solution 4

Observe that the length of the area where the two circles intersect can be found explicitly as .

Let , then the power of point with regards to the larger circle gives
Problem
The adjoining figure shows two intersecting chords in a circle, with on minor arc . Suppose that the
radius of the circle is , that , and that is bisected by . Suppose further that is the
only chord starting at which is bisected by . It follows that the sine of the minor arc is a rational
number. If this fraction is expressed as a fraction in lowest terms, what is the product ?

Solution
Let be any fixed point on circle and let be a chord of circle . The locus of midpoints of the
chord is a circle , with diameter . Generally, the circle can intersect the chord at two
points, one point, or they may not have a point of intersection. By the problem condition, however, the
circle is tangent to BC at point N.

Let M be the midpoint of the chord . From right triangle , .


Thus, .

Notice that the distance equals (Where is the radius of


circle P). Evaluating this, . From , we see

that

Next, notice that . We can therefore apply the tangent subtraction formula
to obtain , . It follows

that , resulting in an answer of .

Das könnte Ihnen auch gefallen